PT8.S1.Q5 - A person can develop or outgrow asthma...

ironofffironofff Member
edited May 2021 in Logical Reasoning 70 karma

Can someone please explain how did we get C as correct AC. We don't have to have same totals in order to get same percentages. Why does this question requires us to make this assumption? Thank you!

Comments

  • canihazJDcanihazJD Alum Member Sage
    8460 karma

    boys <10 2x likely to get it
    boys less likely to outgrow it
    lots of girls get it in early adolescence
    same % boys/girls by adolescence

    Therefore # adolescent boys w/it = # adolescent girls w/it

    So boys start off with a higher percentage and the disparity is maintained until it closes as a result of an increase in girls getting asthma in early adolescence.

    If you know there is a similar # in each population, you could reasonably conclude the ending #s would be close as well.

    Why does this question requires us to make this assumption?

    This is not a required assumption... remember that would be a necessary assumption. Here we are asked to find something that leads us to the conclusion provided in the stem, a sufficient assumption. Its a pretty loose answer... very MSS-like, which you see in a lot of the earlier content, but the others are trash.

  • ironofffironofff Member
    70 karma

    Thank you so Much!

Sign In or Register to comment.